Nature de deux intégrales

etanche
Modifié (May 2022) dans Analyse
Bonjour
Convergente ou divergente ?
$$
1) \quad \int_{0}^{1}\exp\Big(\sum_{k=0}^{\infty} x^{2^k} \Big) dx,
\qquad \qquad
2) \quad \int_{0}^{1}\exp\Big(\sum_{k=0}^{\infty} x^{3^k} \Big) dx.

$$ Merci.

Réponses

  • Problème de Mathematics magazine si je ne m'abuse pas.
  • Si l'intégrale du 1) était convergente on en déduirait $e\leq 2$.

    Mais pour celle du 2) la condition $e\leq3$ n'est pas une contradiction.
  • @ $e \leq 2$ peux-tu détailler merci
  • Pour la première intégrale, on peut procéder ainsi.

    Soit $\lambda\gg1.$ Pour $n\gg1,$ si $\displaystyle 1-\frac{1}{\lambda 2^{n}}\leq x\leq 1-\frac{1}{\lambda 2^{n+1}}$ alors :
    \begin{align*}
    \sum\limits_{k\geq 0}x^{2^k} & \geq \sum\limits_{k=0}^{n}\Big(1-\frac{1}{\lambda 2^{n}}\Big)^{2^k}\\
    & \geq n\Big(1-\frac{1}{\lambda 2^{n}}\Big)^{2^{n}}\\
    & \geq (1-\varepsilon)n \mbox{ car } \Big(1-\frac{1}{\lambda 2^{n}}\Big)^{2^{n}}\sim e^{-\frac{1}{\lambda}}.\\
    \end{align*} D'où
    \begin{align*}
    I_{1} & :=\int_{0}^{1}\exp\Big(\sum\limits_{k\geq 0}x^{2^{k}}\Big)dx\\
    & \geq \sum_{n\geq 0}\int_{1-\tfrac{1}{\lambda 2^{n}}}^{1-\tfrac{1}{\lambda 2^{n+1}}}\exp\Big(\sum\limits_{k\geq 0}x^{2^{k}}\Big)dx\\
    & \gtrsim \sum_{n\geq 1} \frac{\exp\big((1-\varepsilon)n\big)}{2^{n}}=+\infty.

    \end{align*} Pour la deuxième intégrale, on procède de la même manière que précédemment (à une comparaison série-intégrale près pour la queue de la série... je ne vais pas écrire les détails !) pour obtenir que $\displaystyle I_{2}:=\int_{0}^{1}\exp\Big(\sum\limits_{k\geq 0}x^{3^{k}}\Big)dx$ est convergente.

    On peut montrer que l'on a au voisinage de $1^{-}$ (en $x$) : $$\sum\limits_{k\geq 0}x^{2^k}\sim \frac{-\ln(1-x)}{\ln(2)} \mbox{ et } \sum\limits_{k\geq 0}x^{3^k}\sim \frac{-\ln(1-x)}{\ln(3)}.
    $$ Ces équivalents permettent également de conclure facilement.
  • @BobbyJoe avec les deux équivalents que tu donnes au voisinage de 1 à savoir -ln(1-x)/ln(2). et -ln(1-x)/ln(3) ça montre que les deux intégrales sont divergentes, non ?
  • Non car ln(2) est strictement plus petit que 1 et ln(3) est strictement plus grand que 1 (bien penser à utiliser le critère de l’oncle Ben’s : riz-man :0).
  • Puisque BobbyJoe a proposé une solution détaillée pour la première intégrale je propose la mienne.

    La fonction $f_2 : x\mapsto \displaystyle\sum_{k=0}^{\infty} x^{2^k}$ vérifie pour $0\leq x<1$ : $f_2(x)=x+f_2(x^2)$.

    Le changement de variable $x=t^2$ donne $I_{1}=\displaystyle\int_{0}^{1}\exp f_2(x) dx=\int_0^1 2te^{-t} \exp f_2(t)dt$.

    Comme $0\leq te^{-t}\leq e^{-1}$ on obtient $e\leq2$ si $I_1$ converge.


    Je n'ai pas trouvé aussi simple pour $I_2$, j'ai fait comme BobbyJoe dans sa deuxième méthode.

    Une comparaison série-intégrale permet de démontrer que, pour $a>1$, on a au voisinage de $1$ :

    $f_a(x)=\displaystyle\sum_{k=0}^{\infty} x^{a^k}\sim \int_0^{+\infty}\exp(a^t\ln x) dt=\dfrac1{\ln a}\int_{\ln(-\ln x)}^{+\infty}\exp(-e^u)du$ par $u=t\ln a+\ln(-\ln x)$.

    On en déduit $f_a(x)\sim \dfrac{-\ln(-\ln x)}{\ln a}\sim \dfrac{-\ln(1-x)}{\ln a}$ puis la convergence de $I_a=\displaystyle\int_{0}^{1}\exp f_a(x) dx$ quand $a>e$ et sa divergence quand $a<e$.
Connectez-vous ou Inscrivez-vous pour répondre.